Difference between revisions of "2015 AMC 12B Problems/Problem 18"

(Solution)
 
(2 intermediate revisions by 2 users not shown)
Line 15: Line 15:
  
 
==Solution 2==
 
==Solution 2==
 +
Think backwards. The range is the same as the numbers <math>y</math> that can be expressed as the sum of two or more prime positive integers.
  
Think backwards. The range is the same as the numbers <math>y</math> that can be expressed as the sum of two or more prime positive integers.
+
The lowest number we can get is <math>y = 2+2 = 4</math>. For any number greater than 4, we can get to it by adding some amount of 2's and then possibly a 3 if that number is odd. For example, 23 can be obtained by adding 2 ten times and adding a 3; this corresponds to the argument <math>n = 2^{10} \times 3</math>.  Thus our answer is <math>\boxed{\textbf{(D)}\; \text{the set of integers greater than }3}</math>.
  
The lowest number we can get is <math>y = 2+2 = 4</math>. For any number greater than <math>4</math>, we can get to it by adding some amount of <math>2</math>'s and then possibly a <math>3</math> if that number is odd. For example, <math>23</math> can be obtained by adding <math>2</math> 10 times and adding a <math>3</math>; this corresponds to the argument <math>n = 2^10 \cdot 3</math>. Thus our answer is <math>\boxed{\textbf{(D)}\; \text{the set of integers greater than }3}</math>.
+
==Solution 3==
 +
Notice that by the Chicken McNugget Theorem, given <math>2</math>s and <math>3</math>s, we can make any number larger than <math>2\cdot3 - 2 - 3 = 1.</math> We also exclude <math>2</math> and <math>3</math> since they correspond to primes. All other numbers can be written as <math>2a + 3b,</math> which <math>r(2^a3^b)</math> will equal. Thus the answer is <math>D.</math>
  
 
==See Also==
 
==See Also==
 
{{AMC12 box|year=2015|ab=B|num-a=19|num-b=17}}
 
{{AMC12 box|year=2015|ab=B|num-a=19|num-b=17}}
 
{{MAA Notice}}
 
{{MAA Notice}}

Latest revision as of 03:03, 3 August 2021

Problem

For every composite positive integer $n$, define $r(n)$ to be the sum of the factors in the prime factorization of $n$. For example, $r(50) = 12$ because the prime factorization of $50$ is $2 \times 5^{2}$, and $2 + 5 + 5 = 12$. What is the range of the function $r$, $\{r(n): n \text{ is a composite positive integer}\}$ ?

$\textbf{(A)}\; \text{the set of positive integers} \\ \textbf{(B)}\; \text{the set of composite positive integers} \\ \textbf{(C)}\; \text{the set of even positive integers} \\ \textbf{(D)}\; \text{the set of integers greater than 3} \\ \textbf{(E)}\; \text{the set of integers greater than 4}$

Solution

Solution 1

This problem becomes simple once we recognize that the domain of the function is $\{4, 6, 8, 9, 10, 12, 14, 15, \dots\}$. By evaluating $r(4)$ to be $4$, we can see that $\textbf{(E)}$ is incorrect. Evaluating $r(6)$ to be $5$, we see that both $\textbf{(B)}$ and $\textbf{(C)}$ are incorrect. Since our domain consists of composite numbers, which, by definition, are a product of at least two positive primes, the minimum value of $r(n)$ is $4$, so $\textbf{(A)}$ is incorrect. That leaves us with $\boxed{\textbf{(D)}\; \text{the set of integers greater than }3}$.

Solution 2

Think backwards. The range is the same as the numbers $y$ that can be expressed as the sum of two or more prime positive integers.

The lowest number we can get is $y = 2+2 = 4$. For any number greater than 4, we can get to it by adding some amount of 2's and then possibly a 3 if that number is odd. For example, 23 can be obtained by adding 2 ten times and adding a 3; this corresponds to the argument $n = 2^{10} \times 3$. Thus our answer is $\boxed{\textbf{(D)}\; \text{the set of integers greater than }3}$.

Solution 3

Notice that by the Chicken McNugget Theorem, given $2$s and $3$s, we can make any number larger than $2\cdot3 - 2 - 3 = 1.$ We also exclude $2$ and $3$ since they correspond to primes. All other numbers can be written as $2a + 3b,$ which $r(2^a3^b)$ will equal. Thus the answer is $D.$

See Also

2015 AMC 12B (ProblemsAnswer KeyResources)
Preceded by
Problem 17
Followed by
Problem 19
1 2 3 4 5 6 7 8 9 10 11 12 13 14 15 16 17 18 19 20 21 22 23 24 25
All AMC 12 Problems and Solutions

The problems on this page are copyrighted by the Mathematical Association of America's American Mathematics Competitions. AMC logo.png